Вы находитесь на странице: 1из 17

1

EE370/01 Examination No. 1 Fall 2008/09


(50 minutes)

Kuwait University
Electrical Engineering Department







Name :

Student I. D. : .

Signature : .



Problem No. Grade
1 25

2 25

3 25

4 25

Total

100














2
Problem 1 (25 points): Drive the expression for
) (
) (
) (
s R
s U
s M = in terms of ) (s G
i
for the following
system:














Problem 2 (25 points): Can you approximate the step response
) 10 ( ) (
) 5 . 10 (
) (
2
+ + +
+
=
s b s a s s
s
s C as
the step response of 2
nd
order system with
5
2
damping ratio and 2 seconds settling time? If yes,
find the values of a and b? Justify your answer.
Problem 3 (25 points): Use the Routh Table to find the range of K that keeps the following
feedback system stable.







Problem 4 (25 points): (a) Find the steady state error, ) ( = t e , in terms of a and b for the
following system:










(b) Find the values of a and b that yield zero steady state error.








+
+
+
_

_


C(s)
R(s)
+
_



+
+
+
_


+
+

3




EE370/01 Examination No. 1 Fall 2006/07

Kuwait University
Electrical Engineering Department







Name :

Student I. D. : .

Signature : .



Problem No. Grade
1 20

2 20

3 20

4 20

5

20
Total

100













4



Problem 1 (20 points): Can you use the partial fraction expansion
2
4 3
2
2
2
1
) 2 ( 2 1 1 +
+
+
+
+
+
+ s
k
s
k
s
s k
s
k

to find the inverse Laplace transform of
2 2
2
) 2 )( 1 (
3
+ +
+
s s
s
if:
a) 0
1
= k
b) 0
3
= k
c)
3 2
k k =
Explain your answer.




3 2 4
0 4 4
1 2 4
0
) 2 4 ( ) 4 4 ( ) 2 4 ( ) (
) 1 ( ) 2 )( 1 ( ) 2 ( ) 2 ( 3
4 3 1
3 2 1
4 3 2 1
3 2
4 3 1 3 2 1
2
4 3 2 1
3
3 2
2
4
2
3
2
2
2
1
2
= + +
= + +
= + + +
= +
+ + + + + + + + + + + =
+ + + + + + + + = +
k k k
k k k
k k k k
k k
k k k s k k k s k k k k s k k
s k s s k s s k s k s



a) No, you cannot solve the equations
b) No, you cannot solve the equations
c) Yes, you can solve the equations.

















5




Problem 2 (20 points):

a) (10 points): Drive the expression of the output ) (s C in terms of
( ), ( ), ( ), ( ), ( ) M s H s G s R s and D s for the following system:











b) (10 points): What conditions on ( ) M s and ) (s G you will impose if you want to make the
steady state output ) (t c equals zero when 0 ) ( = s R and ) (s D is a step function.





LHP the in GM of roots the all plus
M d
M G
H G
s
d
GM
H G
s C b
D
GM
H G
R
GM
G
C HD MC R HD G
HD GX C a
s ss
+
= =
+
+
=
+
+
=
+
+
+
+
= + + =
+ =

1
) 0 ( 0
) 0 ( ) 0 ( 1
) 0 ( )) 0 ( 1 (
1
) 1 (
lim )
1
) 1 (
1
) (
)




















+
- +
+
+
+
-
6


Problem 3 (20 points): Consider the following closed-loop system:








Find the values of
1
k and
2
k to get 0.5 settling time.








ion approximat order nd ensure to k
k
k w
T
k w
k k s k s
k s
s s
k s
k
s s
k s
s T
n
s
n
2 0
14 5 . 0
2
8 4
2 2
) 10 ( ) 2 (
10 2
1
10 2
) (
1
2
2
2
2 1 2
2
1
2
1
2
2
1
>>
= =
+
= =
+ =
+ + + +
+
=
+ +
+
+
+ +
+
=
,
,




















C(s)
R(s)
+
_
7



Problem 4 (20 points): Use Routh-Hurwitz criterion to find the values of
0
c ,
1
c ,
2
c that will cause
the polynomial
0 1
2
2
3 4
) ( c s c s c s s s M + + + + = to have two real roots in the LHP, one real root in
the RHP, and one root on the je -axis.




4
s
1
2
c
0
c
3
s
1
1
c
2
s
1 2
c c
0
c
1
s
1 2
0 1 1 2 1
c c
c c c c c





0
s
0
c



Make row s
2
equal zero:
1 2
c c = and 0
0
= c
Form the polynomial from s c s row s
1
3 3
: +
Differentiate the polynomial:
1
2
3 c s +










You have 3 symmetrical roots, select 0
1
< c (one sign change): one root on the RHP and its
symmetrical root on the LHP plus one root on jw-axis. Also, one root on the LHP since you have
4 roots in total.

Then 0
1 2
< = c c and 0
0
= c









4
s
1
2
c
0
c
3
s
1
1
c
2
s
3
1
c
1
s
1
1 1
3
2
3
3
c
c c
=




0
s
1
c
8



Problem 5 (20 points): A unity negative feedback system has the open loop transfer function

2
1
) (
2
2 1
+
+
=
s s
s k k
s G
. Determine the gains
1
k and
2
k that minimize the steady state error
due to the step, ramp, or parabolic input.





) (
2
) 2 (
lim ) (
2
1
1
lim ) (
) ( 1
lim ) (
1 2
2 3
3
0
2
2 1
0 0
s R
k s k s s
s s
s R
s s
s k k
s
s R
s G
s
t e
s s s
+ + +
+
=
+
+
+
=
+
= =




Then 0 5 . 0
1 2
> > k k for stability

























3
s
1
2
k
2
s
2
1
k
1
s
1 2
5 . 0 k k

0
s
1
k

9

EE370/01 Examination no. 1 Fall 2005/06

Kuwait University
Electrical Engineering Department







Name :

Student I.D. :.




Problem no. Grade
1 20

2 20

3 20

4 20

5

20
Bonus

20
Total

120















10
Problem 1 (20 points): Given the following differential equation:


2
2
( ) ( ) ( 0.01)
3 2 ( ) ( 0.01).
d y t d y t d u t
y t u t
dt dt dt

+ + = +

Use Laplace transform to find ( ) y t for a unit step input ( ) u t and zero initial conditions.


Problem 2 (20 points): Find the value of 0 a > that will cause the following system to have some
poles on the je -axis. Use Routh-Hurwitz criterion.



Problem 3 (20 points):
c) (10 points): Drive the expression of the error ( ) ( ) ( ) E s R s C s = in terms of
( ), ( ), ( ), ( ), ( ) M s H s G s R s and D s for the following stable system:












d) (10 points): If ( ) R s and ( ) D s are unit step functions, find the relation between
( ) H s and ( ) M s in order to get zero steady state error.













+
-






+
- +
+
+
+
11





Problem 4 (20 points): Consider the following closed-loop system:

Can you find the values of
1
0 k > and
2
0 k > to meet the following specifications:
Closed-loop damping ratio 0.8 , = .
Closed-loop settling time 10
s
T = sec.
Steady state error of 0.1 due to a unit step input.
Write the reason if you cannot find
1
k and
2
k .


Problem 5 (20 points):
a) (16 points): Sketch the root locus for the following system.

Show the following on the root locus:
Breakaway and/or break-in points.
Angles of departure and/or arrival.
b) (4 points): Can you find 0 k > to stabilize the closed-loop system.


Bonus Problem (20 points): Consider the following feedback system








where 7 1 a s s . Find the value of 0 k > that will minimize the steady state error due to a unit
step input for
C(s)



R(s)
+ +
- -


+
-


+
-
12
EE370/01 Examination no. 1 Spring 2006

Kuwait University
Electrical Engineering Department







Name :

Student I.D. :.




Problem no. Grade
1 25

2 25

3 25

4 25

Bonus

20
Total

120


















13

Problem 1 (25 points): Given the closed-loop transfer function
2
( ) 3 4
( ) :
( ) 12 12
Y s s
T s
R s s s
+
= =
+ +
for the
following system









e) (10 points): Determine the closed-loop poles and zeros. Also, show if there is any dominant
pole(s).
f) (15 points): Find the transfer function ( ) G s ?


Problem 2 (25 points): Given the following system
a) (10
points): Determine the value of 0 k > that will minimize the steady state error
due to a unit step input ( ) r t .
b) (15 points): Determine the value of 0 k > that will minimize the sensitivity of the error ( ) E s
to a change in the parameter a .

Problem 3 (25 points): Given
3 2
( ) 6 M s s s ks c = + + + , find the range of k in term of c so that all
the roots of ( ) M s have a negative real part. Use Routh-Hurwitz criterion.

Problem 4 (25 points): Consider the system:







Determine the values of , , a b c and k to have a unit step response ( ) y t with: steady state value of 2,
maximum amplitude of 3 and settling time of 4 sec.



+
-


2
+
+
+
-


k
+
-


14
Bonus (20 points): Given the Laplace transform of
( ( 1)) d f a t
dt

equals G(s), find the


Laplace transform of ( ) f t in term of G(s)?


EE370/51 Examination No. 1 Spring 2009
(50 minutes)

Kuwait University
Electrical Engineering Department







Name (Arabic) :

Student I. D. : .

Signature : .



Problem No. Grade
1 25

2 25

3 25

4 25

Total

100












15




Problem 1 (25 points): For the following system, find ) (s U ?

















































+
+
+
_

_

_

16



Problem 2 (25 points): Consider the following translational mechanical system. A 2 N force f(t)
is applied for 0 > t , find the values of
v
f , M, and K such that the response x(t) is given by the
plot shown below.

























0 0.5 2
0
1
1.1
Response
x

(
t
)
Step Response
Time (sec)
A
m
p
l i t
u
d
e
M
f
v
K
f (t)
x (t)
17



Problem 3 (25 points): Use the Routh Table to find the range of o that makes the following
feedback system unstable.








Problem 4 (25 points): (a: 15 points) Derive the expression for the error ) ( ) ( ) ( s C s R s E = in term
of ) (s H for the following system:








(b: 10 points) Find the value of ) 0 ( = s H that minimizes the steady state error.












C(s) R(s)
+
_



+
_
+
+

Вам также может понравиться